fix(geometria): corregge la seconda parte degli ultimi appunti sul prodotto scalare

main
parent 856a184446
commit 8df0b7c301

@ -252,19 +252,19 @@ forma bilineare simmetrica, ossia un prodotto scalare su $\KK^n$.
\end{proof}
\begin{definition}
Si definisce \textbf{congruenza} la relazione di equivalenza $\sim$ definita nel seguente
Si definisce \textbf{congruenza} la relazione di equivalenza $\cong$ definita nel seguente
modo su $A, B \in M(n, \KK)$:
\[ A \sim B \iff \exists P \in GL(n, \KK) \mid A = P^\top A P. \]
\[ A \cong B \iff \exists P \in GL(n, \KK) \mid A = P^\top A P. \]
\end{definition}
\begin{remark}
Si può facilmente osserva che la congruenza è in effetti una relazione di equivalenza. \\
Si può facilmente osservare che la congruenza è in effetti una relazione di equivalenza. \\
\li $A = I^\top A I \implies A \sim A$ (riflessione), \\
\li $A \sim B \implies A = P^\top B P \implies B = (P^\top)\inv A P\inv = (P\inv)^\top A P\inv \implies B \sim A$ (simmetria), \\
\li $A \sim B \implies A = P^\top B P$, $B \sim C \implies B = Q^\top C Q$, quindi $A = P^\top Q^\top C Q P =
(QP)^\top C (QP)$ (transitività).
\li $A = I^\top A I \implies A \cong A$ (riflessione), \\
\li $A \cong B \implies A = P^\top B P \implies B = (P^\top)\inv A P\inv = (P\inv)^\top A P\inv \implies B \cong A$ (simmetria), \\
\li $A \cong B \implies A = P^\top B P$, $B \cong C \implies B = Q^\top C Q$, quindi $A = P^\top Q^\top C Q P =
(QP)^\top C (QP) \implies A \cong C$ (transitività).
\end{remark}
\begin{remark}

@ -18,7 +18,8 @@
\begin{note}
Nel corso del documento, per $V$ si intenderà uno spazio vettoriale di dimensione
finita $n$ e per $\varphi$ un suo prodotto scalare.
finita $n$ e per $\varphi$ un suo prodotto scalare. Analogamente si intenderà lo stesso
per $V'$ e $\varphi'$.
\end{note}
\begin{proposition} (formula delle dimensioni del prodotto scalare)
@ -133,11 +134,93 @@
di elementi nulli.
\end{remark}
\begin{definition} (somma diretta ortogonale)
Siano i sottospazi $U$ e $W \subseteq V$ in somma diretta. Allora si dice che $U$ e $W$ sono in \textbf{somma
diretta ortogonale rispetto al prodotto scalare} $\varphi$ di $V$, ossia che $U \oplus W = U \oplus^\perp W$, se $\varphi(\vec u, \vec w) = 0$ $\forall \vec u \in U$, $\vec w \in W$.
\end{definition}
\begin{definition} (cono isotropo)
Si definisce \textbf{cono isotropo} di $V$ rispetto al prodotto scalare $\varphi$ il seguente insieme:
\[ \CI(\varphi) = \{ \v \in V \mid \varphi(\v, \v) = 0 \}, \]
\vskip 0.05in
ossia l'insieme dei vettori isotropi di $V$.
\end{definition}
\begin{note}
La notazione $\varphi > 0$ indica che $\varphi$ è definito positivo.
Analogamente $\varphi < 0$ indica che $\varphi$ è definito negativo.
La notazione $\varphi > 0$ indica che $\varphi$ è definito positivo (si scrive $\varphi \geq 0$ se invece è semidefinito
positivo).
Analogamente $\varphi < 0$ indica che $\varphi$ è definito negativo (e $\varphi \leq 0$ indica che è semidefinito negativo).
\end{note}
\begin{exercise} Sia $\Char \KK \neq 2$.
Siano $\vv1$, ..., $\vv k \in V$ e sia $M = \left( \varphi(\vv i, \vv j) \right)_{i, j = 1\textrm{---}k} \in M(k, \KK)$,
dove $\varphi$ è un prodotto scalare di $V$. Sia inoltre $W = \Span(\vv 1, ..., \vv k)$. Si dimostrino
allora le seguenti affermazioni.
\begin{enumerate}[(i)]
\item Se $M$ è invertibile, allora $\vv 1$, ..., $\vv k$ sono linearmente indipendenti.
\item Siano $\vv 1$, ..., $\vv k$ linearmente indipendenti. Allora $M$ è invertibile $\iff$ $\restr{\varphi}{W}$ è non degenere $\iff$ $W \cap W^\perp = \zerovecset$.
\item Siano $\vv1$, ..., $\vv k$ a due a due ortogonali tra loro. Allora $M$ è invertibile $\iff$ nessun
vettore $\vv i$ è isotropo.
\item Siano $\vv1$, ..., $\vv k$ a due a due ortogonali tra loro e siano anche linearmente indipendenti.
Allora $M$ è invertibile $\implies$ si può estendere $\basis_W = \{\vv 1, \ldots, \vv k\}$ a una base ortogonale di $V$.
\item Sia $\KK = \RR$. Sia inoltre $\varphi > 0$. Allora $\vv 1$, ..., $\vv k$ sono linearmente
indipendenti $\iff$ $M$ è invertibile.
\item Sia $\KK = \RR$. Sia ancora $\varphi > 0$. Allora se $\vv 1$, ..., $\vv k$ sono a due a due
ortogonali e sono tutti non nulli, sono anche linearmente indipendenti.
\end{enumerate}
\end{exercise}
\begin{solution}
\begin{enumerate}[(i)]
\item Siano $a_1$, ..., $a_k \in \KK$ tali che $a_1 \vv 1 + \ldots + a_k \vv k = 0$. Vale in
particolare che $\vec 0 = \varphi(\vv i, \vec 0) = \varphi(\vv i, a_1 \vv 1 + \ldots + a_k \vv k) =
\sum_{j=1}^k a_j \varphi(\vv i, \vv j)$ $\forall 1 \leq i \leq k$. Allora $\sum_{j=1}^k a_j M^j = 0$.
Dal momento che $M$ è invertibile, $\rg(M) = k$, e quindi l'insieme delle colonne di $M$ è linearmente
indipendente, da cui si ricava che $a_j = 0$ $\forall 1 \leq j \leq k$, e quindi che $\vv 1$, ...,
$\vv k$ sono linearmente indipendenti.
\item Poiché $\vv 1$, ..., $\vv k$ sono linearmente indipendenti, tali vettori formano una base di
$W$, detta $\basis$. In particolare, allora, vale che $M = M_\basis(\restr{\varphi}{W})$. Pertanto,
se $M$ è invertibile, $\Rad(\restr{\varphi}{W}) = \Ker M = \zerovecset$, e dunque $\restr{\varphi}{W}$
è non degenere. Se invece $\restr{\varphi}{W}$ è non degenere, $\zerovecset = \Rad(\restr{\varphi}{W}) = W \cap W^\perp$. Infine, se $W \cap W^\perp = \zerovecset$, $\zerovecset = W \cap W^\perp = \Rad(\restr{\varphi}{W}) = \Ker M$, e quindi $M$ è iniettiva, e dunque invertibile.
\item Dal momento che $\vv 1$, ..., $\vv k$ sono ortogonali tra loro, $M$ è una matrice diagonale.
Pertanto $M$ è invertibile se e solo se ogni suo elemento diagonale è diverso da $0$, ossia
se $\varphi(\vv i, \vv i) \neq 0$ $\forall 1 \leq i \leq k$, e dunque se e solo se nessun vettore
$\vv i$ è isotropo.
\item Se $M$ è invertibile, da (ii) si deduce che $\Rad(\restr{\varphi}{W}) = W \cap W^\perp = \zerovecset$,
e quindi che $W$ e $W^\perp$ sono in somma diretta. Inoltre, per la formula delle dimensioni del prodotto
scalare, $\dim W + \dim W^\perp = \dim V + \underbrace{\dim (W \cap V^\perp)}_{\leq \dim (W \cap W^\perp) = 0} = \dim V$. Pertanto $V = W \oplus^\perp W^\perp$. \\
Allora, dacché $\Char \KK \neq 2$, per il teorema di Lagrange, $W^\perp$ ammette una base ortogonale $\basis_{W^\perp}$. Si conclude
dunque che $\basis = \basis_W \cup \basis_{W^\perp}$ è una base ortogonale di $V$.
\item Se $M$ è invertibile, da (i) $\vv1$, ..., $\vv k$ sono linearmente indipendenti. Siano ora
invece $\vv 1$, ..., $\vv k$ linearmente indipendenti per ipotesi. Siano $a_1$, ..., $a_k \in \KK$ tali
che $a_1 M^1 + \ldots + a_k M^k = 0$, allora $a_1 \varphi(\vv i, \vv 1) + \ldots + a_k \varphi(\vv i, \vv k) = 0$ $\forall 1 \leq i \leq k$. Pertanto, detto $\v = a_1 \vv 1 + \ldots + a_k \vv k$, si ricava che:
\[ \varphi(\v, \v) = \sum_{i=1}^k \sum_{j=1}^k a_j \, \varphi(\vv i, \vv j) = 0. \]
Tuttavia questo è possibile solo se $\v = a_1 \vv 1 + \ldots + a_k \vv k = 0$. Dal momento che
$\vv 1$, ..., $\vv k$ sono linearmente indipendenti, si conclude che $a_1 = \cdots = a_k = 0$, ossia
che le colonne di $M$ sono tutte linearmente indipendenti e quindi che $\rg(M) = k \implies$ $M$ è invertibile.
\item Poiché $\vv 1$, ..., $\vv k$ sono ortogonali a due a due tra loro, $M$ è una matrice diagonale.
Inoltre, dacché $\varphi > 0$ e $\vv i \neq \vec 0$ $\forall 1 \leq i \leq k$, gli elementi diagonali di $M$ sono sicuramente tutti diversi da zero, e quindi $\det (M) \neq 0$ $\implies$ $M$ è invertibile. Allora,
per il punto (v), $\vv 1$, ..., $\vv k$ sono linearmente indipendenti.
\end{enumerate}
\end{solution}
\begin{definition}
Data una base ortogonale $\basis$ di $V$ rispetto al prodotto
scalare $\varphi$,
@ -146,7 +229,7 @@
\begin{align*}
\iota_+(\varphi) &= \max\{ \dim W \mid W \subseteq V \E \restr{\varphi}{W} > 0 \}, &\text{(}\textbf{indice di positività}\text{)} \\
\iota_-(\varphi) &= \max\{ \dim W \mid W \subseteq V \E \restr{\varphi}{W} < 0 \}, &\text{(}\textbf{indice di negatività}\text{)}\\
\iota_0(\varphi). &= \dim V^\perp &\text{(}\textbf{indice di nullità}\text{)}
\iota_0(\varphi) &= \dim V^\perp &\text{(}\textbf{indice di nullità}\text{)}
\end{align*}
Quando il prodotto scalare $\varphi$ è noto dal contesto, si omette
@ -221,52 +304,67 @@
ortogonale $\basis$ con forma quadratica nulla, si osserva che $W = \Span(\ww 1, ..., \ww k)$ altro non è che $V^\perp$ stesso. Infatti, come
visto anche nella dimostrazione del teorema di Sylvester reale, vale
che $\dim W = \dim \Ker (M_\basis(\varphi)) = \dim V^\perp$. Inoltre,
se $\w \in W$ e $\v \in V$, $\varphi(\w, \v) = \varphi(\sum_{i=1}^k \alpha_i \ww i, \sum_{i=1}^k \beta_i \ww i + \sum_{i=k+1}^n \beta_i \vv i) = \sum_{i=1}^k \alpha_i \beta_i q(\ww i) = 0$, e quindi
se $\w \in W$ e $\v \in V$, $\varphi(\w, \v) = \varphi(\sum_{i=1}^k
\alpha_i \ww i, \sum_{i=1}^k \beta_i \ww i + \sum_{i=k+1}^n \beta_i \vv i)
= \sum_{i=1}^k \alpha_i \beta_i q(\ww i) = 0$, e quindi
$W \subseteq V^\perp$, da cui si conclude che $W = V^\perp$. \\
\li Vale in particolare che $\rg(\varphi) = \iota_+ + \iota_-$, mentre
$\dim \Ker(\varphi) = \iota_0$, e quindi $n = \iota_+ + \iota_- + \iota_0$.
\end{remark}
\begin{definition}
\begin{definition} (isometria)
Dati due spazi vettoriali $(V, \varphi)$ e
$(V', \varphi')$ dotati di prodotto scalare sullo stesso campo $\KK$, si dice che
$V$ e $V'$ sono \textbf{isometrici} se esiste un isomorfismo
$f$, detto isometria, che preserva tali che prodotti, ossia tale che:
$f$, detto \textit{isometria}, che preserva tali che prodotti, ossia tale che:
\[ \varphi(\vec v, \vec w) = \varphi'(f(\vec v), f(\vec w)). \]
\end{definition}
\begin{exercise}\nl
\li $f : V \to V'$ è un isometria $\iff$ per una base $\basis =
\{\vv1, ..., \vv k\}$ di $V$, $\varphi(\vv i, \vv j) = \varphi'(f(\vv i), f(\vv j))$ $\iff$ vale per ogni base.
\begin{exercise} Sia $f : V \to V'$ un isomorfismo. Allora $f$ è un'isometria $\iff$ $\forall$ base $\basis = \{ \vv 1, \ldots, \vv n \}$ di $V$, $\basis' = \{ f(\vv 1), \ldots, f(\vv n) \}$ è una base di $V'$ e $\varphi(\vv i, \vv j) = \varphi'(f(\vv i), f(\vv j))$ $\forall 1 \leq i, j \leq n$ $\iff$ $\exists$ base $\basis = \{ \vv 1, \ldots, \vv n \}$ di $V$ tale che $\basis' = \{ f(\vv 1), \ldots, f(\vv n) \}$ è una base di $V'$ e $\varphi(\vv i, \vv j) = \varphi'(f(\vv i), f(\vv j))$ $\forall 1 \leq i, j \leq n$.
\end{exercise}
\begin{proposition}
Per $(V, \varphi)$ e $(V', \varphi')$ sono equivalenti:
\begin{solution} Se $f$ è un'isometria, detta $\basis$ una base di $V$, $\basis' = f(\basis)$ è una base di $V'$
dal momento che $f$ è anche un isomorfismo. Inoltre, dacché $f$ è un'isometria, vale sicuramente che
$\varphi(\vv i, \vv j) = \varphi'(f(\vv i), f(\vv j))$ $\forall 1 \leq i, j \leq n$. \\
Sia ora assunto per ipotesi che $\forall$ base $\basis = \{ \vv 1, \ldots, \vv n \}$ di $V$, $\basis' = \{ f(\vv 1), \ldots, f(\vv n) \}$ è una base di $V'$ e $\varphi(\vv i, \vv j) = \varphi'(f(\vv i), f(\vv j))$ $\forall 1 \leq i, j \leq n$. Allora, analogamente a prima, detta $\basis = \{ \vv 1, \ldots, \vv n \}$ una base di $V$, $\basis' = f(\basis)$ è una base di $V'$, e in quanto tale,
per ipotesi, è tale che $\varphi(\vv i, \vv j) = \varphi'(f(\vv i), f(\vv j))$ $\forall 1 \leq i, j \leq n$. \\
Sia infine assunto per ipotesi che $\exists$ base $\basis = \{ \vv 1, \ldots, \vv n \}$ di $V$ tale che $\basis' = \{ f(\vv 1), \ldots, f(\vv n) \}$ è una base di $V'$ e $\varphi(\vv i, \vv j) = \varphi'(f(\vv i), f(\vv j))$ $\forall 1 \leq i, j \leq n$. Siano $\v$, $\w \in V$. Allora $\exists a_1$, ..., $a_n$, $b_1$, ..., $b_n \in \KK$
tali che $\v = a_1 \vv 1 + \ldots + a_n \vv n$ e $\w = b_1 \vv 1 + \ldots + b_n \vv n$. Si ricava pertanto
che:
\[ \varphi'(f(\v), f(\w)) = \sum_{i=1}^n \sum_{j=1}^n a_i b_j \, \varphi'(f(\vv i), f(\vv j)) =
\sum_{i=1}^n \sum_{j=1}^n a_i b_j \, \varphi(\vv i, \vv j) = \varphi(\v, \w), \]
da cui la tesi.
\end{solution}
\begin{proposition} Sono equivalenti le seguenti affermazioni:
\begin{enumerate}[(i)]
\item $V$ e $V'$ sono isometrici;
\item $\forall$ base $\basis$ di $V$, $\basis'$ di $V'$,
$M_\basis(\varphi)$ e $M_\basis'(\varphi')$ sono congruenti;
\item lo stesso ma per una base.
\item $\forall$ base $\basis$ di $V$, base $\basis'$ di $V'$,
$M_\basis(\varphi)$ e $M_{\basis'}(\varphi')$ sono congruenti;
\item $\exists$ base $\basis$ di $V$, base $\basis'$ di $V'$ tale che
$M_\basis(\varphi)$ e $M_{\basis'}(\varphi')$ sono congruenti.
\end{enumerate}
\end{proposition}
\begin{proof}
(1-2)
\begin{proof} Se $V$ e $V'$ sono isometrici, sia $f : V \to V'$ un'isometria. Sia $\basisC = \{ \vv 1, \ldots, \vv n \}$ una base di $V$. Allora, poiché $f$ è anche un isomorfismo, $\basisC' = f(\basisC)$ è una base di $V$ tale che
$\varphi(\vv i, \vv j) = \varphi'(f(\vv i), f(\vv j))$ $\forall 1 \leq i, j \leq n$. Pertanto $M_\basisC(\varphi) = M_{\basisC'}(\varphi')$. Si conclude allora che, cambiando base in $V$ (o in $V'$), la matrice associata
al prodotto scalare varia per congruenza dalla formula di cambiamento di base per il prodotto scalare, da cui si ricava che per ogni scelta di $\basis$ base di $V$ e di $\basis'$ base di $V'$, $M_\basis(\varphi) \cong M_{\basis'}(\varphi')$. Inoltre, se tale risultato è vero per ogni $\basis$ base di $V$ e di $\basis'$ base di $V'$, dal momento che sicuramente esistono due basi $\basis$, $\basis'$ di $V$ e $V'$, vale anche (ii) $\implies$ (iii). \\
\rightproof Sia $\basis'' = f(\basis)$. Allora $M_{\basis''}(\varphi')=
(\varphi'(f(\vv i), f(\vv j))) = (\varphi(\vv i, \vv j))$. Allora,
per la formula di cambiamento di base, le matrici
$M_{\basis'}(\varphi')$ e $M_{\basis'}(\varphi')$ sono congruenti.
\leftproof Sia $A = M_\basis(\varphi) = P^\top B P$ e
$B = M_{\basis'}(\varphi')$. Allora $a_{ij} = \varphi(\vv i, \vv j) =
... = \varphi'(\vec{v_i^{ii}}, \vec{v_j^{ii}})$, dove
$\vec{v_i^{ii}}$ è base perché $P$ è invertibile. Allora
l'applicazione $f : V \to V'$ che manda $\vv i \mapsto \vec{v_{i}^{''}}$ è un isometria.
(2-3) esercizio.
Si dimostra ora (iii) $\implies$ (i). Per ipotesi $M_\basis(\varphi) \cong M_{\basis'}(\varphi')$, quindi
$\exists P \in \GL(n, \KK) \mid M_{\basis'}(\varphi') = P^\top M_\basis(\varphi) P$. Allora $\exists$ $\basis''$
base di $V'$ tale che $P = M_{\basis''}^{\basis'}(\Idv)$, da cui $P\inv = M_{\basis'}^{\basis''}(\varphi)$. Per la formula di cambiamento di base del prodotto
scalare, $M_{\basis''}(\varphi) = (P\inv)^\top M_{\basis'} P\inv = M_\basis(\varphi)$. Detta
$\basis'' = \{ \ww 1, \ldots, \ww n \}$, si costruisce allora l'isomorfismo $f : V \to V'$ tale
che $f(\vv i) = \ww i$ $\forall 1 \leq i \leq n$.. Dal momento che per costruzione $M_\basis(\varphi) = M_{\basis''}(\varphi')$,
$\varphi(\vv i, \vv j) = \varphi'(\ww i, \ww j) = \varphi'(f(\vv i), f(\vv j))$ $\forall 1 \leq i, j \leq n$.
Si conclude dunque che $\varphi(\v, \w) = \varphi'(f(\v), f(\w))$ $\forall \v, \w \in V$, e dunque
che $f$ è un'isometria, come desiderato dalla tesi.
\end{proof}
\begin{proposition} $(V, \varphi)$ e $(V', \varphi')$ spazi vettoriali
@ -274,8 +372,11 @@
isometrici $\iff$ $\varphi$ e $\varphi'$ hanno la stessa segnatura.
\end{proposition}
\begin{proof}
\rightproof Basta che prendi la solita base. \\
\begin{proof}\nl\nl
\rightproof Per la precedente proposizione, esistono due basi $\basis$ e $\basis'$, una di $V$ e una di $V'$,
tali che $M_\basis(\varphi) \cong M_{\basis'}(\varphi)$. Allora queste due matrici condividono la stessa
segnatura. \\
\leftproof Siano $\basis$, $\basis'$ basi di Sylvester di $V$
e di $V'$. Si definisce allora l'applicazione $f : V \to V'$ tale
che $f(\vv i) = \ww i$: essa è un isometria.
@ -286,10 +387,6 @@
la stessa segnatura.
\end{corollary}
\begin{definition} (somma diretta ortogonale)
$V = U \oplusperp W$.
\end{definition}
\begin{remark}\nl
\li se $V = U \oplusperp W$, allora $\iota_+(\varphi) = \iota_+(\restr{\varphi}{V}) + \iota_+(\restr{\varphi}{W})$, e
analogamente per gli altri indici.

@ -14,6 +14,8 @@
\Large \textbf{Esercitazione: forma canonica di Jordan reale}
\end{center}
\wip
\begin{exercise}
Sia $M \in M(n, \RR)$ tale che $\exists a_1$, ..., $a_k \in \RR$ distinti
tale che:

@ -111,6 +111,7 @@
\newcommand{\rvline}{\hspace*{-\arraycolsep}\vline\hspace*{-\arraycolsep}}
\newcommand{\Idv}{Id_V}
\DeclareMathOperator{\CI}{CI}
\DeclareMathOperator{\Bil}{Bil}
\DeclareMathOperator{\Mult}{Mult}
\DeclareMathOperator{\sgn}{sgn}
@ -181,6 +182,8 @@
\newcommand{\basis}{\mathcal{B}}
\newcommand{\BB}{\mathcal{B}}
\newcommand{\basisC}{\mathcal{B}}
\newcommand{\HH}{\mathbb{H}}
\newcommand{\FFp}[1]{\mathbb{F}_p}

Loading…
Cancel
Save